Duda sobre la Ley Circuital de Ampere

La ley del circuito de Ampere establece

B d   =   m 0 I

Podemos usarlo para derivar fácilmente el campo magnético de un cable portador de corriente infinitamente largo. Mi pregunta es, ¿por qué el cable debe ser infinitamente largo? Sé que tiene algo que ver con B siendo constante y tangencial al ciclo en cada punto para facilitar la evaluación de la integral, pero no puedo encontrar una explicación a mi pregunta.

Respuestas (2)

Usamos el caso idealizado de una corriente infinitamente larga para poder justificar (por simetría) que la fuerza del campo solo dependerá de la coordenada radial r , por lo que se puede sacar de la integral, ya que solo estamos integrando sobre el ángulo que parametriza un círculo alrededor del alambre:

B d = B 0 2 π r   d θ = 2 π r B = m 0 I B = m 0 I 2 π r

Si el cable no es infinitamente largo, puede moverse hacia el final del mismo, donde es obvio que el B -field no debería depender solo de la coordenada radial, por lo que nuestro cálculo simple falla. En la práctica, a menudo se puede usar este caso ideal como una buena aproximación para el campo cercano al cable, siempre que la distancia desde el cable sea mucho menor que la longitud del cable, el efecto es más o menos el de un cable infinito. .

"donde es obvio que el campo B no debería depender solo de la coordenada radial, por lo que nuestro cálculo simple falla": creo que si aplica la ley de Biot-Savart, también puede ver hacia el final que todavía es tangencial al bucle circular . Y ahí es donde creo que está la pregunta de OP. Creo que la respuesta probable es: no podemos tomar solo un cable de corriente recta finito sin violar la ecuación de continuidad.
@ user22180 Para este sistema, las coordenadas naturales son cilíndricas. Toma el cable para estar a lo largo X = y = 0 r = 0 . Entonces, el campo de un alambre infinito tiene simetría traslacional en el z dirección, pero un alambre finito no. Esto significa que, para un alambre finito, B = B ( r , z ) , por lo que nuestra fórmula simple no puede ser correcta. Está claro que el campo justo al lado del medio de un alambre finito no puede ser el mismo que el campo al lado y encima del alambre.
ver el campo magnético sigue siendo tangencial. Y estoy de acuerdo contigo en que en el caso de alambre finito la magnitud de la B depende de z también. Pero no depende de ϕ lo cual es importante en este caso de integral de línea. Como no depende de ϕ todavía puedes sacar el campo magnético de la integral y tienes B(r,z)= m 0 I 2 π r . Pero esto es inconsistente ya que el lado izquierdo muestra dependencia de Z (así lo es nuestra intuición), pero el lado derecho no muestra dependencia de Z. Creo que esta es la pregunta de OP.
Creo que está obteniendo esta inconsistencia ya que está rompiendo la ecuación de continuidad al tomar solo un cable de corriente finita.
@ user22180 Si bien entiendo su punto ("¿de dónde viene la corriente?"), Simplemente estaba señalando que la independencia de ϕ es irrelevante. Para un alambre finito, B = m 0 I 2 π r es simplemente incorrecto, ya que el campo sobre el cable se debilitará cada vez más, lo que NO se refleja en esta fórmula . Matemáticamente, debemos tener B 0 como | z | para cualquier alambre finito. Entonces, la solución simple es realmente incorrecta y esto no tiene nada que ver con la ecuación de continuidad.
mi duda real era: podemos evaluar la integral fácilmente si B es constante y tangencial al ciclo. Esto sucede en el caso de un cable finito, entonces, ¿por qué no puedo usar la ley de amperios? Sé que está dando un resultado incorrecto, pero ¿por qué?
Sucede por las razones que describí a continuación: uno pierde la invariancia de traducción en el z dirección, por lo que resulta imposible concluir que B = B ( r ) , lo que nos habría llevado a la solución simple. Otra forma de verlo es observar que la corriente encerrada también se convierte en z dependiente en este caso.

En primer lugar, le sugiero que lea los comentarios que he hecho en la respuesta de Danu para verificar si he entendido su pregunta o no.

Ver, B d   =   m 0 I se ha obtenido únicamente sobre la base de × B = m 0 j . Pero en realidad la ecuación de Maxwell es × B = m 0 ( j + ϵ 0 mi t ) lo cual es consistente con la ecuación de continuidad .

Ahora en el caso de alambre infinito × B = m 0 j es suficiente como en la región de interés j = 0 .

Pero en el caso de alambre finito solo hay acumulación de carga en el alambre finito de modo que j + ρ t = 0 . Tan relevante ecuación de Maxwell es × B = m 0 ( j + ϵ 0 mi t ) . Ver la ley de Ampere en este caso está dada por this , no por el B d   =   m 0 I . Entonces, está aplicando una fórmula incorrecta y es por eso que está obteniendo una respuesta incorrecta.